Ist der Magnus-Effekt eine Folge des Bernoulli-Prinzips?

Der Magnus-Effekt wird üblicherweise mit dem Bernoulli-Prinzip erklärt. Am Beispiel des Auftriebs an einem rotierenden Zylinder wird die Geschwindigkeitsdifferenz jedoch durch die zusätzliche Arbeit des rotierenden Zylinders verursacht, nicht jedoch durch die Druckdifferenz. Das Bernoulli-Prinzip ist im Grunde eine Energieeinsparung entlang einer Stromlinie. Allerdings wird hier die Energie nicht aufgrund von externer Arbeit konserviert. Wird das Bernoulli-Prinzip also missbraucht, um den Magnus-Effekt zu erklären?

Es ist keine Energieeinsparung entlang einer Stromlinie. Es ist Impulserhaltung entlang einer Stromlinie. Es ist F = M A , was Impulserhaltung ist.

Antworten (4)

Dies ist eine ausgezeichnete und scharfsinnige Frage. Letztendlich kommt es auf Experimente an: Das folgende Modell funktioniert für viele Flüssigkeiten ziemlich gut. Dies muss also bedeuten, dass der Verlust klein genug ist, dass jedes Flüssigkeitsteilchen beim Vorbeiströmen an der Störungsregion einen Bruchteil seiner Energie verliert, der klein genug ist, um das der Bernoulli-Gleichung zugrunde liegende Energiegleichgewicht nicht stark zu stören . Gleichzeitig ist die Viskosität groß genug, dass der Zylinder die Zirkulation in der Strömung aufrechterhalten kann.

Sobald man eine Zirkulation in einem Fluss hergestellt hat, wird die Zirkulation verweilen – manchmal fast unbegrenzt, mit sehr wenig weiterem Energieaufwand. Die Vorticity-Gleichung zeigt dies. Sie können sich daher eine Situation vorstellen, in der der Zylinder "zufällig" in einer Strömung mit Zirkulation sitzt, ohne sich Gedanken darüber zu machen, wie diese Zirkulation entstanden ist. Man kann dann ein verlustfreies mathematisches Modell entwickeln , das tatsächlich den Magnus-Effekt zeigt, den Sie nach der Tiefe und Scharfsinnigkeit Ihrer Frage vielleicht bereits kennen. In diesem Modell hat die Strömung eine gewisse angenommene Zirkulation und man denkt nicht darüber nach, wie diese Zirkulation zustande gekommen ist. Ich spreche von einer reibungsfreien, drehungsfreien Strömung, deren komplexes Geschwindigkeitspotential ist:

(1) Ω : { z C | | z | > A } C ; Ω ( z ) = A v ( z A + A z ) + Γ 2 π ich Protokoll z

Wo v C ist die Geschwindigkeit des Fluids weit vom Zylinder entfernt ( dh der Zylinder ist von einer anfänglich gleichmäßigen Strömung durchdrungen) und Γ R ist der Kreislauf. Der Querschnitt des Zylinders ist die Region { z C | | z | A } .

Dies ist ein vollkommen stationärer Fluss, und sobald die Zirkulation aufgebaut ist, hält sie sich selbst aufrecht. Es gibt nirgendwo im Modell Verluste, also wenden Sie das Theorem von Blasius an , um den Auftrieb zu berechnen, was einfach die quantitative Berechnung um den Zylinder des Arguments des gewöhnlichen Satzes von Bernoulli ist.

So könnte man sich folgendes Gedankenexperiment vorstellen (nicht praktikabel). Sie haben eine magische Flüssigkeit, deren Viskosität Sie nach Belieben ein- und ausschalten können. Sie haben einen gleichmäßigen Fluss in dieser Flüssigkeit und lassen Ihren Zylinder darin eintauchen, und mit internen Motoren oder was auch immer drehen Sie den Zylinder und die verlustbehaftete Viskosität wird eine Zirkulation in der Strömung aufbauen. Dann schaltest du die Viskosität schlagartig aus. Die Zirkulation wird in der Strömung verweilen und jetzt, in Abwesenheit von Verlusten, können Sie die obige Berechnung durchführen und sehen, dass es tatsächlich einen Auftrieb gibt. Beachten Sie, dass man immer eine Zirkulation benötigt, um einen Auftrieb ungleich Null zu erzeugen.


Frage von OP

Ich stimme Ihnen zu, dass die Zirkulation der Hauptgrund für den Lift ist. Aber das Bernoulli-Argument ist irgendwie fehlerhaft, weil die Geschwindigkeitsdifferenz nicht zu einer Druckdifferenz führt, es ist die Zirkulation, die zu einer Druckdifferenz führt. Das Bernoulli-Prinzip geht immer von keiner Viskosität und Verwirbelung aus. Glauben Sie, dass das Bernoulli-Prinzip sogar in einem heuristischen Argument missbraucht wird?

Es ist immer noch das Bernoulli-Argument, das den Ursprung der Druckgradienten und damit letztlich der Kraft beschreibt. Die Zirkulation führt einfach eine Asymmetrie in die Strömung ein, die dann die Summe der Drücke ungleich Null macht.

Das Blasius-Theorem entspricht dem Bernoulli-Prinzip wie folgt: auf einem Abschnitt D z einer Kontur um den Körper in der komplexen Ebene ist die Druckkraft nach dem Bernoulli-Prinzip:

(2) ich ρ 2 ( | v | 2 | D z Ω | 2 ) D z

Wo v wie in (1) definiert ist und ρ die Flüssigkeitsdichte. Hier wird wie in (1) die Vektorrichtung durch die Phase der komplexen Zahl gezeigt. Die Kontur um den Körperrand ist eine Stromlinie, so dass die Stromfunktion (Imaginärteil des komplexen Potentials) entlang ihr konstant ist. Daher, um den Rand des Körpers, | D z Ω | 2 = ( R e ( D z Ω ) ) 2 = ( D z Ω ) ) 2 so dass wir beim Summieren von (2) um die geschlossene Kontur herum, um die Nettokraft zu finden, mit etwas in der Nähe eines gewöhnlichen Konturintegrals enden (wenn wir das beobachten v 2 D z = 0 ):

(3) F = ich ρ 2 ( D z Ω ) ) 2 D z = π ρ Reste von  ( D z Ω ) ) 2  an den Polen innerhalb des Zylinders

was ohne weiteres ausgearbeitet wird F = ich v ρ Γ so dass F = ich v ρ Γ , also senkrecht zur Strömung. Sie können also sehen, dass das Ergebnis nach dem Bernoulli-Prinzip Ihnen sagt, dass der Auftrieb proportional zur Zirkulation ist.

Ich stimme Ihnen zu, dass die Zirkulation der Hauptgrund für den Lift ist. Aber das Bernoulli-Argument ist irgendwie fehlerhaft, weil die Geschwindigkeitsdifferenz nicht zu einer Druckdifferenz führt, es ist die Zirkulation, die zu einer Druckdifferenz führt. Das Bernoulli-Prinzip geht immer von keiner Viskosität und Verwirbelung aus. Glauben Sie, dass das Bernoulli-Prinzip sogar in einem heuristischen Argument missbraucht wird?
@ user3229471 Siehe meine Ergänzungen oben.
@Shadumu Stimmen Sie meiner Erklärung zu?

Geben Sie hier die Bildbeschreibung ein

Wie in der Abbildung gezeigt, wird das rotierende Wasser im Eimer konkav, und je schneller die Rotation, desto konkaver ist die Wasseroberfläche. Dies zeigt, dass bei einer bestimmten Höhe H der Druck umso geringer ist, je näher das Wasser am Rotationszentrum ist. Und je schneller sich das Wasser dreht, desto geringer ist der Druck des Wassers im Rotationszentrum.

Geben Sie hier die Bildbeschreibung ein

Wie in der Abbildung gezeigt, dreht sich der Luftstrom auf der rechten Seite des Balls langsam, da der Luftstrom auf der rechten Seite des Balls der Rotationsrichtung des Balls entgegengesetzt ist; Der Luftstrom auf der linken Seite des Balls dreht sich in die gleiche Richtung wie die Rotationsrichtung des Balls, sodass der Luftstrom auf der linken Seite des Balls schnell rotiert. Nach der Schlussfolgerung zum Eimer muss der Druck auf der linken Seite des Balls geringer sein als auf der rechten Seite des Balls. Auf den Ball wird also eine von rechts nach links gerichtete Kraft F ausgeübt.

Daher verwende ich das Bernoulli-Prinzip nicht, um den Magnus-Effekt zu erklären.

Ich kann mich irren, aber beweist das Bernoulli-Prinzip nicht, dass sich das Objekt auf die Seite mit dem niedrigeren Druck bewegt, da die rechte Seite einen höheren Druck hat?

Ich bin Flieger und meiner Meinung nach widerspricht der Magnus-Effekt Bernoulli wie folgt: Uns wird beigebracht, dass die Erhöhung der Geschwindigkeit eine Verringerung des Drucks bewirkt. Der sich drehende Zylinder führt zu einer Geschwindigkeitszunahme des Fluids relativ zur Zylinderoberfläche auf der einen Seite und zu einer Abnahme auf der anderen Seite. Die Druckreduzierung sollte daher auf der Seite erfolgen, die sich schneller in die allgemeine Bewegungsrichtung des Zylinders bewegt, was eine Kraft zu der Seite verursacht, die sich in Bewegungsrichtung bewegt. Wir sehen jedoch aus der Beobachtung, dass die Kraft in die entgegengesetzte Richtung wirkt.

Ich bin kein Physiker und ich habe keine Gleichungen, um zu beweisen, was ich sagen werde, und ich könnte mich absolut irren, aber ich habe genau über diese Frage nachgedacht und sie hat mich zu diesem Thread geführt. Ich würde argumentieren, dass das Bernoulli-Prinzip dem Magnus-Effekt entgegengesetzt wirken sollte. Die Rotationsbewegung würde die hohe Geschwindigkeit und den niedrigen Druck auf der gegenüberliegenden Seite erzeugen, wo der Magnus-Effekt seine Kraft entfaltet. Ich schieße Gewehre und habe normalerweise kleine Zylinder, die sich im Uhrzeigersinn von mir wegbewegen (Rechtsdrehung). Bei einem Wind von links nach rechts würde der Bernoulli-Effekt dazu führen, dass die Luft über die Oberseite des Projektils beschleunigt wird und einen leichten Auftrieb verursacht. Es hat sich gezeigt, dass der Magnus-Effekt unter diesen Bedingungen tatsächlich einen negativen Auftrieb verursacht. Ich denke an den Magnus-Effekt wie an eine sich drehende Billardkugel. Wenn der Ball mit Spin auf die Stoßstange trifft, Es wird von der Wand abprallen und einen Teil seines Spins in Bewegung in Richtung des Spins umwandeln. Vielleicht liege ich daneben, aber so sehe ich das.

Es hat sich gezeigt, dass der Magnus-Effekt unter diesen Bedingungen tatsächlich einen negativen Auftrieb verursacht . Ein Zitat oder Link für diese Behauptung wäre für Ihre Antwort hilfreich.